21
$\begingroup$

For $\gamma$ an ordinal, let “$H_\gamma$” be the statement:

For all ordinals $\alpha$, we have $2^{\aleph_\alpha} = \aleph_{\alpha+\gamma}$.

So clearly $H_0$ is false, and so is $H_\omega$; in fact, $H_\gamma$ implies that $\gamma$ is successor (because otherwise $\operatorname{cf}\gamma = \omega_\alpha$, say, and $\aleph_{\alpha+\gamma} = 2^{\aleph_\alpha} = (2^{\aleph_\alpha})^{\aleph_\alpha} = (\aleph_{\alpha+\gamma})^{\aleph_\alpha} = (\aleph_{\alpha+\gamma})^{\operatorname{cf}\aleph_{\alpha+\gamma}}$ gives a contradiction).

On the other hand, $H_1$ is precisely the generalized continuum hypothesis (GCH) and it is consistent relative to ZFC.

I understand from this question and this one that $H_2$ is known to be consistent relative to certain large cardinal assumptions, and perhaps even $H_k$ for any concrete $k < \omega$.

What else, if anything, is known about the consistency of the various $H_\gamma$? Might we perhaps construct¹ a successor ordinal $\gamma$ for which $H_\gamma$ is demonstrably false?

  1. Admittedly, I don't know how to phrase this question properly, because clearly “the smallest successor ordinal for which $H_\gamma$ is false” is a definable successor ordinal for which $H_\gamma$ is provably false, which is clearly not what I'm asking about. But a proof in ZFC that $H_{\omega+1}$ is false (say) would be a good answer to my question.
$\endgroup$

1 Answer 1

21
$\begingroup$
  1. By a result of Patai, $\gamma$ should be finite (this is exercise 5.15 in Jech's book).

  2. For any finite $n>0, H_n$ is consistent, see Merimovich's paper A power function with a fixed finite gap everywhere.


For completeness and since not everyone has Jech's book at hand, here's how the proof of (1) goes (note that Jech uses "$\beta$" in place of "$\gamma$"). Suppose $\gamma\ge \omega$ is such that $2^{\aleph_\eta}=\aleph_{\eta+\gamma}$ for all $\eta$. Letting $\alpha$ be minimal such that $\alpha+\gamma>\gamma$ we have $0<\alpha\le\gamma$ and $\alpha$ is a limit. Consider $\kappa=\aleph_{\alpha\cdot 2}$. This $\kappa$ is obviously singular and by choice of $\alpha$ we have $$2^{\aleph_{\alpha+\xi}}=\aleph_{\alpha+\xi+\gamma}=\aleph_{\alpha+\gamma}$$ for each $\xi<\alpha$. From this we get $$2^\kappa=\aleph_{\alpha+\gamma}$$ (more generally, if $\lambda$ is singular and $\mu=\max\{2^\theta:\theta<\lambda\}$ exists then $2^\lambda=\mu$). On the other hand, we have $$2^\kappa=\aleph_{\alpha\cdot 2+\gamma}=\aleph_{\alpha+(\alpha+\gamma)}>\aleph_{\alpha+\gamma}$$ again by choice of $\alpha$. So we have a contradiction.

$\endgroup$
0

Your Answer

By clicking “Post Your Answer”, you agree to our terms of service and acknowledge you have read our privacy policy.

Not the answer you're looking for? Browse other questions tagged or ask your own question.